Question

A circular area with a radius of 6.00 cm lies in the xy-plane. You may want to review (Page) For related problemsolving tips

A circular area with a radius of 6.00 cm lies in the xy-plane. 

You may want to review (Page) 

For related problemsolving tips and strategies, you may want to view a Video Tutor Solution of Magnetic flux calculations. 

Part A What is the magnitude of the magnetic flux through this circle due to a uniform magnetic field with a magnitude of 0.220 T in the + z-direction? 

Part B What is the magnitude of the magnetic flux through this circle due to the same magnetic field (with a magnitude of 0.220 T ), now at an angle of 53.8 from the + z-direction? 

Part C What is the magnitude of the magnetic flux through this circle due to the same magnetic field (with a magntiude of 0.220 T ), now in the + y -direction? 

1 0
Add a comment Improve this question Transcribed image text
Answer #1

\(A\) )

The magnetic flux through the circle is

$$ \begin{aligned} \phi_{\mathrm{B}} &=B A \cos 0^{\circ} \\ &=(0.220 \mathrm{~T})\left(\pi\left(6.00 \times 10^{-2} \mathrm{~m}\right)^{2}\right) \\ &=2.488 \times 10^{-3} \mathrm{~T} \cdot \mathrm{m}^{2}=2.49 \times 10^{-3} \mathrm{~T} \cdot \mathrm{m}^{2} \end{aligned} $$

B) The magnetic flux through the circle is

$$ \begin{aligned} \phi_{\mathrm{B}} &=B A \cos 53.8^{\circ} \\ &=(0.220 \mathrm{~T})\left(\pi\left(6.00 \times 10^{-2} \mathrm{~m}\right)^{2}\right) \cos 53.8^{\circ} \\ &=1.47 \times 10^{-3} \mathrm{~T} \cdot \mathrm{m}^{2} \end{aligned} $$

C)

The magnetic flux through the circle is

$$ \begin{aligned} \phi_{\mathrm{B}} &=B A \cos 90^{\circ} \\ &=0 \end{aligned} $$

> May I ask why it has to be multiplied to pi?

T1N4CH01 Thu, Jun 24, 2021 4:55 AM

Add a comment
Know the answer?
Add Answer to:
A circular area with a radius of 6.00 cm lies in the xy-plane. You may want...
Your Answer:

Post as a guest

Your Name:

What's your source?

Earn Coins

Coins can be redeemed for fabulous gifts.

Not the answer you're looking for? Ask your own homework help question. Our experts will answer your question WITHIN MINUTES for Free.
Similar Homework Help Questions
  • A circular area with a radius of 7.40 cm lies in the x-y plane.

    A circular area with a radius of 7.40 cm lies in the x-y plane.For related problem-solving tips and strategies, you may want to view a Video Tutor Solution of Magnetic flux.Part AWhat is the magnitude of the magnetic flux through this circle due to a uniform magnetic field B =0.247 T that points in the +z direction?Part BWhat is the magnitude of the magnetic flux through this circle due to a uniform magnetic field B =0.247 T that points at...

  • A circular area with a radius of 6.5 cm lies in the xy plane...

    A circular area with a radius of 6.5 cm lies in the xy plane. What is the magnitude of the magnetic flux through this circle due to a uniform magnetic field B=0.230 T:(A) in the +z direction;(B) at an angle of 53.1 degrees from the +z direction;(C) in the +y direction?

  • A circular area with a radius of 5.70 cm lies in the xy-plane. What is the...

    A circular area with a radius of 5.70 cm lies in the xy-plane. What is the magnitude of the magnetic flux through this circle due to a uniform magnetic field B = 0.185 T oriented in the following ways? (a) in the +z-direction _________ Wb (b) at an angle of 53.1

  • A circular area with a radius of 7.70 cm lies in the x-y plane. What is...

    A circular area with a radius of 7.70 cm lies in the x-y plane. What is the magnitude of the magnetic flux through this circle due to a uniform magnetic field B = 0.212 T that points at an angle of 52.6 ∘ from the+z direction? What is the magnitude of the magnetic flux through this circle due to a uniform magnetic field B = 0.212 T that points in the +y direction?

  • A circular area with a radius of 7.20 cm lies in the x-y plane. What is...

    A circular area with a radius of 7.20 cm lies in the x-y plane. What is the magnitude of the magnetic flux through this circle due to a uniform magnetic field B = 0.212 T that points at an angle of 52.6 ∘ from the +z direction?

  • A circular loop of wire with a radius of 15 cm lies in the xy plane...

    A circular loop of wire with a radius of 15 cm lies in the xy plane and carries a current of 1.9 A, counterclockwise. It is placed in an external magnetic field of the form B = 4ị mt. Z y The magnitude and direction of the net torque are: Select one: O a. 47.75 E-3 Nm, -x direction b. 47.75 E-3 Nm, +x direction O c. 47.75 E-3 Nm, +z direction O d. 310.39 E-3 Nm, +x direction e....

  • A loop of wire with radius R lies in the xy-plane and carries a time varying...

    A loop of wire with radius R lies in the xy-plane and carries a time varying current I(t) I, sin(ot). A smaller loop of wire with radius a lies in the xy-plane at the center of the larger loop. Assume that radius of the smaller loop is much smaller that the radius of the larger loop, a<<lR a) (10 points) At the center of the large loop calculate the magnitude and direction of the magnetic field B(t) due to the...

  • A circular loop of wire with a radius of 15 cm lies in the xy plane...

    A circular loop of wire with a radius of 15 cm lies in the xy plane and carries a current of 1.9 A, counterclockwise. It is placed in an external magnetic field of the form B = 4; mt. -X · The magnitude and direction of the net torque are: Select one O a 0.54 E-3 Nm, - direction Ob.5372.14 E 3 Nm, 4x direction O c.0.54 E-3 Nm, +x direction O d. 0.54 E-3 Nm, z direction e. 5372.14...

  • u Review IP A circular loop with a radius of 4.0 cm lies in the x-y...

    u Review IP A circular loop with a radius of 4.0 cm lies in the x-y plane. The magnetic field in this region of space is uniform and given by B = 0.40 T )x+( -0.10 Tỷ+( 0.53 T ) 2. Part A What is the magnitude of the magnetic flux through this loop? Express your answer using two significant figures. VAE ? m Wb Submit Request Answer Part B Suppose we now increase the component of B, leaving the...

  • A circular loop of wire with area A lies in the xy-plane. As viewed along the...

    A circular loop of wire with area A lies in the xy-plane. As viewed along the z-axis looking in the −z-direction toward the origin, a current I is circulating clockwise around the loop. The torque produced by an external magnetic field B⃗  is given by τ⃗ =D(3i^−3j^), where D is a positive constant, and for this orientation of the loop the magnetic potential energy U=−μ⃗ ⋅B⃗  is negative. The magnitude of the magnetic field is B0=15D/IA. Determine the component Bx of B⃗...

ADVERTISEMENT
Free Homework Help App
Download From Google Play
Scan Your Homework
to Get Instant Free Answers
Need Online Homework Help?
Ask a Question
Get Answers For Free
Most questions answered within 3 hours.
ADVERTISEMENT
ADVERTISEMENT
ADVERTISEMENT